2014 dxdy logo

Научный форум dxdy

Математика, Физика, Computer Science, Machine Learning, LaTeX, Механика и Техника, Химия,
Биология и Медицина, Экономика и Финансовая Математика, Гуманитарные науки


Правила форума


В этом разделе нельзя создавать новые темы.

Если Вы хотите задать новый вопрос, то не дописывайте его в существующую тему, а создайте новую в корневом разделе "Помогите решить/разобраться (М)".

Если Вы зададите новый вопрос в существующей теме, то в случае нарушения оформления или других правил форума Ваше сообщение и все ответы на него могут быть удалены без предупреждения.

Не ищите на этом форуме халяву, правила запрещают участникам публиковать готовые решения стандартных учебных задач. Автор вопроса обязан привести свои попытки решения и указать конкретные затруднения.

Обязательно просмотрите тему Правила данного раздела, иначе Ваша тема может быть удалена или перемещена в Карантин, а Вы так и не узнаете, почему.



Начать новую тему Ответить на тему На страницу Пред.  1, 2
 
 Re: Доказать асимптотическую формулу
Сообщение23.12.2015, 10:35 


15/12/15
20
А разве не так при $\lambda\to\infty$?
или следует взять $x = O(\lambda)$?
Но тут ведь тоже будет $x = 2\ln(\lambda) + O()$

 Профиль  
                  
 
 Re: Доказать асимптотическую формулу
Сообщение23.12.2015, 11:15 
Заслуженный участник


08/04/08
8556
B@R5uk в сообщении #1084934 писал(а):
krupen в сообщении #1084930 писал(а):
$x = O(\ln(\lambda^2))$
Это вы плохо сделали. Если собрались искать второй член, то первый надо точно выписать, а не по порядку величины.
Да пофигу. Он же уже нашел оттуда:
krupen в сообщении #1084930 писал(а):
$x = ln(\lambda^2 + ln(O\frac{(ln^2(\lambda^2))}{\lambda^2} + 1)) = 2ln(\lambda) + O\frac{(ln^2(\lambda^2))}{\lambda^2}$
Можно начинать хоть с $x=O(e^{e^{\lambda^e}})$

krupen в сообщении #1084930 писал(а):
Первый член:
...
Второй член:
...
Вы здесь делаете одно и то же действие (разложение логарифма в ряд) 2 раза. Не тратьте зря силы.
Вам уже советовали сделать сразу такое:

Евгений Машеров в сообщении #1082529 писал(а):
$x=\ln(x^2+\lambda^2)=2\ln\lambda+\ln(1+x^2/\lambda^2)=2\ln\lambda+\frac {x^2}{\lambda^2}-\frac {x^4}{2\lambda^4}+\frac {x^6}{3\lambda^6}-\frac {x^8}{4\lambda^8}+\dots$
Вот сюда поставьте 1-е приближение.

Или еще проще:
krupen в сообщении #1084930 писал(а):
krupen в сообщении #1084930 писал(а):
$x = ... = 2\ln(\lambda) + \ln(\frac{4ln^2(\lambda)}{\lambda^2} + \frac{4\ln(\lambda)}{\lambda^2}\cdot  O\frac{(\ln^2(\lambda^2))}{\lambda^2} + O\frac{(\ln^2(\lambda^2))}{\lambda^6} + 1)$


Я знаю, что второй член должен получиться $\frac{4ln^2(\lambda)}{\lambda^2}$, но как его вытащить из скобок, не могу понять
У Вас в конце стоит $\ln (\text{бесконечно-малая}+1)$. Примените формулу $\ln (1+t)=t+O(t^2)$, потом $O$ склейте в одну и все.

Вот это вот: $\frac{4\ln(\lambda)}{\lambda^2}\cdot  O\frac{(\ln^2(\lambda^2))}{\lambda^2}$ упростите сначала, преобразуйте в выражение вида $O(f(\lambda))$. Вы почти решили.

 Профиль  
                  
 
 Re: Доказать асимптотическую формулу
Сообщение23.12.2015, 11:47 


15/12/15
20
Ну если делать таким способом,
Евгений Машеров в сообщении #1082529 писал(а):
Может, так?
$x=\ln(x^2+\lambda^2)=2\ln\lambda+\ln(1+x^2/\lambda^2)=2\ln\lambda+\frac {x^2}{\lambda^2}-\frac {x^4}{2\lambda^4}+\frac {x^6}{3\lambda^6}-\frac {x^8}{4\lambda^8}+\dots$


то получается значительно проще. Но если я, конечно, все делаю правильно, но получается же $x = 2\ln(\lambda) + \frac{4\ln^2(\lambda)}{\lambda^2} - \frac{8\ln^4(\lambda)}{\lambda^4} + O(\frac{1}{\lambda^5})$

Но судя по всему опять не то, потому что в ответе $x = 2\ln(\lambda) + \frac{4\ln^2(\lambda)}{\lambda^2} - \frac{8\ln^4(\lambda) - 16\ln^2(\lambda)}{\lambda^4} + O(\frac{1}{\lambda^5})$

 Профиль  
                  
 
 Re: Доказать асимптотическую формулу
Сообщение23.12.2015, 11:57 
Заслуженный участник
Аватара пользователя


11/03/08
9540
Москва
А подставить следующее приближение?
$x = 2\ln(\lambda) + \frac{4\ln^2(\lambda)}{\lambda^2} + O(\frac{1}{\lambda^3})$

 Профиль  
                  
 
 Re: Доказать асимптотическую формулу
Сообщение23.12.2015, 12:19 


15/12/15
20
Т.е. сначала подставляем $x = 2\ln(\lambda)$ в $x=2\ln\lambda+\frac {x^2}{\lambda^2}-\frac {x^4}{2\lambda^4}+\frac {x^6}{3\lambda^6}-\frac {x^8}{4\lambda^8}+\dots$ и получаем $x = 2\ln(\lambda) + \frac{4\ln^2(\lambda)}{\lambda^2} + O(\frac{1}{\lambda^3})$ и это опять подставляем туда же, что и первый член?

 Профиль  
                  
 
 Re: Доказать асимптотическую формулу
Сообщение23.12.2015, 14:02 
Заслуженный участник


08/04/08
8556
krupen в сообщении #1084971 писал(а):
Т.е. сначала подставляем $x = 2\ln(\lambda)$ в $x=2\ln\lambda+\frac {x^2}{\lambda^2}-\frac {x^4}{2\lambda^4}+\frac {x^6}{3\lambda^6}-\frac {x^8}{4\lambda^8}+\dots$ и получаем $x = 2\ln(\lambda) + \frac{4\ln^2(\lambda)}{\lambda^2} + O(\frac{1}{\lambda^3})$ и это опять подставляем туда же, что и первый член?
Да, именно так. В этом и состоит прикол метода.
У Вас в правую часть $x$ входит поделенный как минимум на $\lambda^2$, потому при каждой подстановке точность будет увеличиваться на $\lambda^2$.

 Профиль  
                  
 
 Re: Доказать асимптотическую формулу
Сообщение23.12.2015, 14:13 


15/12/15
20
а если подставляя $x = 2\ln(\lambda) + \frac{4\ln^2(\lambda)}{\lambda^2} + O(\frac{1}{\lambda^3})$ в $x=2\ln\lambda+\frac {x^2}{\lambda^2}-\frac {x^4}{2\lambda^4}+\frac {x^6}{3\lambda^6}-\frac {x^8}{4\lambda^8}+\dots$ получаются какие-то нереальные степени у логарифма это норм? Просто помимо 3-его члена еще куча всего есть и куда это деть?

 Профиль  
                  
 
 Re: Доказать асимптотическую формулу
Сообщение23.12.2015, 21:17 
Заслуженный участник


08/04/08
8556
krupen в сообщении #1085023 писал(а):
а если подставляя $x = 2\ln(\lambda) + \frac{4\ln^2(\lambda)}{\lambda^2} + O(\frac{1}{\lambda^3})$ в $x=2\ln\lambda+\frac {x^2}{\lambda^2}-\frac {x^4}{2\lambda^4}+\frac {x^6}{3\lambda^6}-\frac {x^8}{4\lambda^8}+\dots$ получаются какие-то нереальные степени у логарифма это норм?
Нереальные - это какие? Видите ли, я задачу не решал :-) (и вряд ли ее здесь кроме Вас кто-то полностью решал). Напишите. пожалуйста.
По-видимому, асимптотическое разложение будет иметь примерно такой вид:
$$x=2\ln \lambda + \sum\limits_{k=1}^{m}\dfrac{P_{2k}(\ln \lambda)}{\lambda^{2k}}+O\left(\dfrac{\ln^{2m} \lambda}{\lambda^{2m}}\right)$$, где $P_{2k}$ - это какие-то многочлены степени $2k$.

krupen в сообщении #1085023 писал(а):
Просто помимо 3-его члена еще куча всего есть и куда это деть?
да как бы не должно ничего страшного быть - максимум 3-4 слагаемых вида $c\dfrac{\ln^r\lambda}{\lambda^4},r=0,...,4$.

 Профиль  
                  
Показать сообщения за:  Поле сортировки  
Начать новую тему Ответить на тему  [ Сообщений: 23 ]  На страницу Пред.  1, 2

Модераторы: Модераторы Математики, Супермодераторы



Кто сейчас на конференции

Сейчас этот форум просматривают: нет зарегистрированных пользователей


Вы не можете начинать темы
Вы не можете отвечать на сообщения
Вы не можете редактировать свои сообщения
Вы не можете удалять свои сообщения
Вы не можете добавлять вложения

Найти:
Powered by phpBB © 2000, 2002, 2005, 2007 phpBB Group